Difference between revisions of "2007 AMC 12A Problems/Problem 6"

(See also)
(Solution)
Line 5: Line 5:
 
<math>\mathrm{(A)}\ 20\qquad \mathrm{(B)}\ 30\qquad \mathrm{(C)}\ 40\qquad \mathrm{(D)}\ 50\qquad \mathrm{(E)}\ 60</math>
 
<math>\mathrm{(A)}\ 20\qquad \mathrm{(B)}\ 30\qquad \mathrm{(C)}\ 40\qquad \mathrm{(D)}\ 50\qquad \mathrm{(E)}\ 60</math>
  
==Solution==
+
==Solution 1==
 
[[Image:2007_AMC12A-6.png]]
 
[[Image:2007_AMC12A-6.png]]
  
Line 12: Line 12:
 
* <math>BAC=\frac{180-40}{2} = 70</math>
 
* <math>BAC=\frac{180-40}{2} = 70</math>
 
* <math>BAD=BAC-DAC=50\ \mathrm{(D)}</math>
 
* <math>BAD=BAC-DAC=50\ \mathrm{(D)}</math>
 +
 +
==Solution 2==
 +
[[File:Example.png]]
  
 
==See also==
 
==See also==

Revision as of 19:14, 10 August 2014

The following problem is from both the 2007 AMC 12A #6 and 2007 AMC 10A #8, so both problems redirect to this page.

Problem

Triangles $ABC$ and $ADC$ are isosceles with $AB=BC$ and $AD=DC$. Point $D$ is inside triangle $ABC$, angle $ABC$ measures 40 degrees, and angle $ADC$ measures 140 degrees. What is the degree measure of angle $BAD$?

$\mathrm{(A)}\ 20\qquad \mathrm{(B)}\ 30\qquad \mathrm{(C)}\ 40\qquad \mathrm{(D)}\ 50\qquad \mathrm{(E)}\ 60$

Solution 1

2007 AMC12A-6.png

We angle chase, and find out that:

  • $DAC=\frac{180-140}{2} = 20$
  • $BAC=\frac{180-40}{2} = 70$
  • $BAD=BAC-DAC=50\ \mathrm{(D)}$

Solution 2

Example.png

See also

2007 AMC 12A (ProblemsAnswer KeyResources)
Preceded by
Problem 5
Followed by
Problem 7
1 2 3 4 5 6 7 8 9 10 11 12 13 14 15 16 17 18 19 20 21 22 23 24 25
All AMC 12 Problems and Solutions
2007 AMC 10A (ProblemsAnswer KeyResources)
Preceded by
Problem 7
Followed by
Problem 9
1 2 3 4 5 6 7 8 9 10 11 12 13 14 15 16 17 18 19 20 21 22 23 24 25
All AMC 10 Problems and Solutions

The problems on this page are copyrighted by the Mathematical Association of America's American Mathematics Competitions. AMC logo.png